Download as pdf or txt
Download as pdf or txt
You are on page 1of 31

www.puucho.

com

PHYSICS
TARGET IIT JEE 2010
XII (ABCD)

MAGNETIC EFFECT
OF CURRENT
CONTENTS
KEY CONCEPT ............................................................. Page –2
EXERCISE–I .................................................................. Page –5
EXERCISE–II ................................................................ Page –8
EXERCISE–III ............................................................... Page –11
OBJECTIVE QUESTION BANK.................................. Page –17
ANSWER KEY ............................................................... Page –29

"GAURAV TOWER" A-10, Road No.-1, I.P.I.A., Kota-324005 (Raj.) INDIA.


Tel.:(0744)2423738, 2423739, 2421097, 2424097, 2423244 Fax: 2436779
92-LIC Colony, Vaishali Nagar, Ajmer (Raj.) Tel.: 0145-2633456
BANSAL CLASSES, Pooja Tower, 3 Gopalpura, Gopalpura Bypass, Jaipur Tel.: 0141-2721107, 2545066
Email: [email protected] Website : www.bansaliitjee.com

Study More with www.puucho.com


www.puucho.com

KEY CONCEPTS
1. A static charge produces only electric field and only electric field can exert a force on it.
A moving charge produces both electric field ans magnetic field and both electric field and magnetic field
can exert force on it.
A current carrying conductor produces only magnetic field and only magnetic field can exert a force on it.
2. Magnetic charge (i.e. current) , produces a magnetic field . It can not produce electric field as net charge
on a current carrying conductor is zero. A magnetic field is detected by its action on current carrying

conductors (or moving charges) and magnetic needles (compass) needles. The vector quantity B known
as MAGNETIC INDUCTION is introduced to characterise a magnetic field . It is a vector quantity which may
be defined in terms of the force it produces on electric currents . Lines of magnetic induction may be
drawn in the same way as lines of electric field . The number of lines per unit area crossing a small area
 
perpendicular to the direction of the induction bring numerically equal to B . The number of lines of B

crossing a given area is referred to as the MAGNETIC FLUX linked with that area. For this reason B is also
called MAGNETIC FLUX DENSITY .
3 MAGNETIC INDUCTION PRODUCED BY A CURRENT (BIOT-SAVART LAW):
The magnetic induction dB produced by an element dl carrying a current I at a distance r is given by :
 

 0  r Idsin    I d x r
or dB
0 r
dB = ,
4 r 2 4 r3
here the quantity Idl is called as current element strength.
 = permeability of the medium = 0 r ; 0 = permeability of free space
r = relative permeability of the medium (Dimensionless quantity).
Unit of 0 & is NA–2 or Hm–1 ; 0 = 4  × 10–7 Hm–1
4 MAGNETIC INDUCTION DUE TO A MOVING CHARGE

 0qv sin 
dBP 
4r 2
  q( v xr )
In vector form it can be written as dB  0
4 r 3
5. MAGNETIC INDUCTION DUE TO AN INIFINITE ST. CONDUCTOR
 0I
B=
2r
6. MAGNETIC INDUCTION DUE TO SEMI INIFINITE ST. CONDUCTOR
 0I
B=
4 r
7. MAGNETIC INDUCTION DUE TO A CURRENT CARRYING STRAIGHT CONDUCTOR
 I
B = 0 (cos 1 + cos 2)
4R
0I
If the wire is very long 1 2  0º then , B =
2R
8. MAGNETIC FIELD DUE TO A FLAT CIRCULAR COIL CARRYING A CURRENT :
 0 NI
(i) At its centre B= , direction
2R
Where N = total number of turns in the coil
I = current in the coil
R = Radius of the coil
 0 NIR 2
(ii) On the axis B=

2 x 2 R 2 3/ 2

Where x = distance of the point from the centre . It is maximum at the centre .
Magnetics Effect of Current [2]

Study More with www.puucho.com


www.puucho.com

9. MAGNETIC INDUCTION DUE TO FLAT CIRCULAR ARC


 I
B= 0
4R
10. MAGNETIC INDUCTION DUE TO SOLENOID
B = 0nI, direction along axis.
where n  no. of turns per m.
I  current
11. MAGNETIC INDUCTION DUE TO TOROID
B = 0nI
N
where n = (no. of turns per m)
2R
N = total turns R >> r
12. MAGNETIC INDUCTION DUE TO CURRENT CARRYING SHEET
1
B = 0I
2
where I = Linear current density (A/m)
13. MAGNETIC INDUCTION DUE TO THICK SHEET
1
At point P2 Bout = 0Id
2
At point P1 Bin = 0Jx
14. GILBERT'S MAGNETISM (EARTH'S MAGNETIC FIELD) :
(a) The line of earth's magnetic induction lies in a vetical plane coinciding with the magnetic North - South
direction at that place. This plane is called the MAGNETIC MERIDIAN. Earth's magnetic axis is slightly
inclined to the geometric axis of earth and this angle varies from 10.50 to 200. The Earth's Magnetic
poles are opposite to the geometric poles i.e. at earth's north pole, its magnetic south pole is situated and
vice versa.
(b) On the magnetic meridian plane , the magnetic induction vector of the earth at any point, generally inclined

to the horizontal at an angle called the MAGNETIC DIP at that place , such that B = total magnetic induction
of the earth at that point.
 
B v = the vertical component of B in the magnetic meridian plane = B sin  .

BH = the horizontal component of B in the magnetic meridian plane = B cos  .
Bv
= tan  .
BH
(c) At a given place on the surface of the earth , the magnetic meridian and the geographic meridian may
not coincide . The angle between them is called "DECLINATION AT THAT PLACE" .
 
15 AMPERES LAW  B . d     I
 I = algebric sum of all the currents .
16. LORENTZ FORCE : 
An electric charge 'q' moving with a velocity V through a magnetic field of magnetic
    
induction B experiences a force F , given by F  qV x B . There fore, if the charge moves
in

a space where both electric and magnetic fields are
 superposed
  .
F = nett electromagnetic force on the charge = q E  q V  B
This force is called the LORENTZ FORCE .
17. MOTION OF A CHARGE IN UNIFORM MAGNETIC FIELD :
  
(a) When v is || to B : Motion will be in a st. line and F = 0
  mv
(b) When v is | to B : Motion will be in circular path with radius R = and angular
qB
Magnetics Effect of Current [3]

Study More with www.puucho.com


www.puucho.com

qB
velocity  = and F = qvB.
 m  mv sin 
(c) When v is at  to B : Motion will be helical with radius Rk = and pitch
qB
PH = 2 mv cos  and F = qvBsin.
qB
18. MAGNETIC FORCE ON A STRAIGHT CURRENT CARRYING WIRE :
  
F  I (L  B)
I = current in the straight conductor

L = length of the conductor in the direction of the current in it

B = magnetic induction. (Uniform throughout the length of conduction)
  
Note : In general force is F   I (d  B)
19. MAGNETIC INTERACTION FORCE BETWEEN TWO PARALLEL LONG STRAIGHT CURRENTS :
When two long straight linear conductors are parallel and carry a current in each , they
magnetically interact with each other , one experiences a force. This force is of :
(i) Repulsion if the currents are anti-parallel (i.e. in opposite direction) or
(ii) Attraction if the currents are parallel (i.e. in the same direction)
 II
This force per unit length on either conductor is given by F = 0 1 2 . Where r = perpendicular
2 r
distance between the parallel conductors
20. MAGNETIC TORQUE ON A CLOSED CURRENT CIRCUIT :
When a plane closed current circuit of 'N' turns and of area 'A' per turn carrying a current
I is placed in uniform magnetic field , it experience a zero nett force , but experience a
    
torque given by   NI A  B  M  B  BINA sin 

When A = area vector outward from the face of the circuit where the current is anticlockwise,
 
B = magnetic induction of the uniform magnetic feild. M = magnetic moment of the current
circuit = IN A

Note : This expression can be used only if B is uniform otherwise calculus will be used.
21. MOVING COIL GALVANOMETER :
It consists of a plane coil of many turns suspended in a radial magnetic feild. when a current
is passed in the coil it experiences a torque which produces a twist in the suspension. This
deflection is directly proportional to the torque  NIAB = K
I =  K   K = elastic torsional constant of the suspension
 
NAB K
I=C  C= = GALVANOMETER CONSTANT..
NAB
22. FORCE EXPERIENCED BY A MAGNETIC DIPOLE IN A NON-UNIFORM MAGNETIC FIELD :
 B
| F | = M r where M = Magnetic dipole moment.
23. FORCE ON A RANDOM SHAPED CONDUCTOR IN MAGNETIC FIELD

1. Magnetic force on a loop in a uniform B is zero
2. Force experienced by a wire of any shape is equivalent to force on a wire
joining points A & B in a uniform magnetic field .
24. MAGNETIC MOMENT OF A ROTATING CHARGE:
If a charge q is rotating at an angular velocity ,
q
its equivalent current is given as I = & its
2
magnetic moment is M = IR = 1/2qR2.
2

NOTE: The rate of magnetic moment to Angular momentum of a uniform rotating object which is charged
uniformly is always a constant. Irrespective of the shape of conductor M/L = q/2m

Magnetics Effect of Current [4]

Study More with www.puucho.com


www.puucho.com

EXERCISE # I
Q.1 A system of long four parallel conductors whose sections with the plane of the
drawing lie at the vertices of a square there flow four equal currents. The directions
of these currents are as follows :
those marked  point away from the reader, while those marked with a dot
point towards the reader. How is the vector of magnetic induction directed at
the centre of the square?
z
y
Q.2 A long straight wire carries a current of 10 A directed along the negative
y-axis as shown in figure. A uniform magnetic field B0 of magnitude 106 T
is directed parallel to the x-axis. What is the resultant magnetic field at the
x
following points?
(a) x = 0 , z = 2 m ; (b) x = 2 m, z = 0 ; (c) x = 0 , z =  0.5 m i

Q.3 Figure shows a straight wire of length l carrying a current i. Find the
magnitude of magnetic field produced by the current at point P.

5 5
Q.4 Two circular coils A and B of radius cm and 5 cm respectively carry current 5 Amp and Amp
2 2
respectively. The plane of B is perpendicular to plane of A and their centres coincide. Find the magnetic
field at the centre.

Q.5 Find the magnetic field at the centre P of square of side a shown in figure.

Q.6 What is the magnitude of magnetic field at the centre ‘O’ of loop of radius 2 m
made of uniform wire when a current of 1 amp enters in the loop and taken out of
it by two long wires as shown in the figure.

Q.7 Find the magnetic induction at the origin in the figure shown.

Q.8 Find the magnetic induction at point O, if the current carrying


wire is in the shape shown in the figure.

Q.9 Find the magnitude of the magnetic induction B of a magnetic field generated
by a system of thin conductors along which a current i is flowing at a
point A (O, R, O), that is the centre of a circular conductor of radius R.
The ring is in yz plane.

Q.10 Two circular coils of wire each having a radius of 4 cm and 10 turns have a common axis and are 6 cm
apart . If a current of 1 A passes through each coil in the opposite direction find the magnetic induction.
(i) At the centre of either coil (ii) At a point on the axis, midway between them.

Magnetics Effect of Current [5]

Study More with www.puucho.com


www.puucho.com

Q.11 Six wires of current I1 = 1A, I2 = 2A, I3 = 3A, I4 = 1A, I5 = 5A and


I6 = 4A cut the page perpendicularly at the points 1, 2, 3, 4, 5 and 6
 
respectively as shown in the figure. Find the value of the integral  B.d l
around the closed path.

Q.12 A cylindrical conductor of radius R carries a current along its length . The current density J, however, it
is not uniform over the cross section of the conductor but is a function of the radius according to J = br,
where b is a constant. Find an expression for the magnetic field B.
(a) at r1 < R & (b) at distance r2 > R, mesured from the axis

Q.13 Electric charge q is uniformly distributed over a rod of length l. The rod is placed parallel to a long wire
carrying a current i. The separation between the rod and the wire is a. Find the force needed to move the
rod along its length with a uniform velocity v.

Q.14 An electron moving with a velocity 5 × 106 ms–1 î in the uniform electric field of 5 × 107 Vm–1 ĵ . Find
the magnitude and direction of a minimum uniform magnetic field in tesla that will cause the electron to
move undeviated along its original path.

Q.15 A charged particle (charge q, mass m) has velocity v0 at origin in +x direction. In space there is a uniform
magnetic field B in - z direction. Find the y coordinate of particle when is crosses y axis.

Q.16 A proton beam passes without deviation through a region of space where there are uniform transverse
mutually perpendicular electric and magnetic field with E and B. Then the beam strikes a grounded
target. Find the force imparted by the beam on the target if the beam current is equal to I.

Q.17 A conducting circular loop of radius r carries a constant current i. It is placed in a uniform magnetic field
 
B0 such that B0 is perpendicular to the plane of the loop. Find the magnetic force acting on the loop.

Q.18 An arc of a circular loop of radius R is kept in the horizontal plane and a
constant magnetic field B is applied in the vertical direction as shown in the
figure. If the arc carries current I then find the force on the arc.

Q.19 A rectangular loop of wire is oriented with the left corner at the origin, one edge
along X-axis and the other edge along Y-axis as shown in the figure. A magnetic
field is into the page and has a magnitude that is given by  = y where  is
contant. Find the total magnetic force on the loop if it carries current i.

Q.20 A particle of charge +q and mass m moving under the influence of a uniform electric field E i and a
magnetic field B k enters in I quadrant of a coordinate system at a point (0, a) with initial velocity v i
and leaves the quadrant at a point (2a, 0) with velocity – 2v j . Find
(a) Magnitude of electric field
(b) Rate of work done by the electric field at point (0, a)
(c) Rate of work done by both the fields at (2a, 0).

Magnetics Effect of Current [6]

Study More with www.puucho.com


www.puucho.com

Q.21 A square current carrying loop made of thin wire and having a mass m =10g can
rotate without friction with respect to the vertical axis OO1, passing through the centre
of the loop at right angles to two opposite sides of the loop. The loop is placed in
a homogeneous magnetic field with an induction B = 10-1 T directed at right angles
to the plane of the drawing. A current I = 2A is flowing in the loop. Find the period
of small oscillations that the loop performs about its position of stable equilibrium.

Q.22 An infinitely long straight wire carries a conventional current I as shown in the
figure. The rectangular loop carries a conventional current I' in the clockwise
direction. Find the net force on the rectangular loop.

Q.23 3 infinitely long thin wires each carrying current i in the same direction , are in the x-y plane of
a gravity free space . The central wire is along the y-axis while the other two are along x = ± d.
(i) Find the locus of the points for which the magnetic field B is zero .
(ii) If the central wire is displaced along the z-direction by a small amount & released, show that it will
execute simple harmonic motion . If the linear density of the wires is  , find the frequency of oscillation.

Q.24 Q charge is uniformly distributed over the same surface of a right circular cone
of semi-vertical angle  and height h. The cone is uniformly rotated about its axis
at angular velocity . Calculated associated magnetic dipole moment.

Magnetics Effect of Current [7]

Study More with www.puucho.com


www.puucho.com

EXERCISE # II
Q.1 Four long wires each carrying current I as shown in the figure are placed
at the points A, B, C and D. Find the magnitude and direction of
(i) magnetic field at the centre of the square.
(ii) force per metre acting on wire at point D.

Q.2 A wire loop carrying current I is placed in the X-Y plane as shown
in the figure
(a) If a particle with charge +Q and mass m is placed at the centre P
and given a velocity along NP (fig). Find its instantaneous acceleration

(b) If an external uniform magnetic induction field B = B i is applied,
find the torque acting on the loop due to the field.

Q.3 A stationary, circular wall clock has a face with a radius of 15cm. Six turns of wire are wound around
its perimeter, the wire carries a current 2.0 A in the clockwise direction. The clock is located, where
there is a constant , uniform external magnetic field of 70 mT (but the clock still keeps perfect time)
at exactly 1:00 pm, the hour hand of the clock points in the direction of the external magnetic field
(a) After how many minutes will the minute hand point in the direction of the torque on the winding due
to the magnetic field ?
(b) What is the magnitude of this torque.

Q.4 A U-shaped wire of mass m and length l is immersed with its two ends
in mercury (see figure). The wire is in a homogeneous field of magnetic
induction B. If a charge, that is, a current pulse q =  idt , is sent through
the wire, the wire will jump up.
Calculate, from the height h that the wire reaches, the size of the charge or current pulse, assuming
that the time of the current pulse is very small in comparision with the time of flight. Make use of the
fact that impulse of force equals  F dt ,which equals mv. Evaluate q for B = 0.1 Wb/m2, m = 10gm,
 = 20cm & h = 3 meters.[g = 10 m/s2]

Q.5 A current i, indicated by the crosses in fig. is established in a strip of copper


of height h and width w. A uniform field of magnetic induction B is applied
at right angles to the strip.

(a) Calculate the drift speed vd for the electrons.

(b) What are the magnitude and dirction of the magnetic force F acting on the
electrons?
(c) What would the magnitude & direction of homogeneous electric field E have
to be in order to counter balance the effect of the magnetic field ?

(d) What is the voltage V necessary between two sides of the conductor in order to create this field E?
Between which sides of the conductor would this voltage have to be applied ?

Magnetics Effect of Current [8]

Study More with www.puucho.com


www.puucho.com

(e) If no electric field is applied form the outside the electrons will be pushed somewhat to one side &
thereforce will give rise to a uniform electric field EH across the conductor untill the force of this electrostatic
field EH balanace the magnetic forces encountered in part (b) . What will be the magnitude and direction
of the field EH? Assume that n, the number of conduction electrons per unit volume, is 1.1x1029/m3
& that h = 0.02 meter , w = 0.1cm , i = 50 amp , & B = 2 webers/meter2.

Q.6(a) A rigid circular loop of radius r & mass m lies in the xy plane on a flat table and has a current
  
I flowing in it. At this particular place , the earth's magnetic field is B  Bx i  By j . How large must
I be before one edge of the loop will lift from table ?
  
(b) Repeat if, B  Bx i  Bz k .

Q.7 A conductor carrying a current i is placed parallel to a current


per unit width j0 and width d, as shown in the figure. Find
the force per unit lenght on the conductor.

Q.8 The figure shows a conductor of weight 1.0 N and length L = 0.5 m placed
on a rough inclined plane making an angle 300 with the horizontal so that conductor
is perpendicular to a uniform horizontal magnetic field of induction B = 0.10
T. The coefficient of static friction between the conductor and the plane is 0.1.
A current of I = 10 A flows through the conductor inside the plane of this
paper as shown. What is the force needed to be the applied parallel to the
inclined plane to sustaining the conductor at rest?

Q.9 An electron gun G emits electron of energy 2kev traveling in the (+)ve
x-direction. The electron are required to hit the spot S where
GS = 0.1m & the line GS makes an angle of 60° with the x-axis,

as shown in the fig. A uniform magnetic field B parallel to GS exists
in the region outsiees to electron gun. Find the minimum value of B
needed to make the electron hit S .

Q.10 Two coils each of 100 turns are held such that one lies in the vertical plane with their centres coinciding.
The radius of the vertical coil is 20 cm and that of the horizontal coil is 30 cm . How would you neutralize
the magnetic field of the earth at their common centre ? What is the current to be passed through each
coil ? Horizontal component of earth's magnetic induction = 3.49 x 10 -5 T and angle of dip = 30º.

Q.11 A uniform, constant magnetic field B is directed at an angle of 45º to the x-axis
in the xy-plane, PQRS is a rigid square wire frame carrying a steady current
I0(clockwise), with its centre at the origin O. At time t = 0, the frame is at
rest in the position shown in the figure, with its sides parallel to the x & y axes.
Each side of the frame is of mass M & length L.

(a) What is the torque  about O acting on the frame due to the magnetic field ?
(b) Find the angle by which the frame rotates under the action of this torque in a short interval of time
t, & the axis about which this rotation occurs (t is so short that any variation in the torque during
this interval may be neglected) Given the moment of inertia of the frame about an axis through its
centre perpendicular to its plane is 4/3 ML2.
Magnetics Effect of Current [9]

Study More with www.puucho.com


www.puucho.com

Q.12 An infinite wire, placed along z-axis, has current I1 in positive z-direction. Aconducting rod placed in xy plane
parallel to y-axis has current I2 in positive y-direction. The ends of the rod subtend + 30° and
– 60° at the origin with positive x-direction. The rod is at a distance a from the origin. Find net force on the rod.

Q.13 A square loop of wire of edge a carries a current i .


(a) Show that B for a point on the axis of the loop and a distance x from its centre is given by,
4  0 ia 2
B= .
  4x 
1/2
 4 x2  a 2 2
 2a 2
(b) Can the result of the above problem be reduced to give field at x = 0 ?
(c) Does the square loop behave like a dipole for points such that x >> a ? If so , what is its dipole moment?

Q.14 A straight segment OC (of length L meter) of a circuit carrying a current


I amp is placed along the x-axis. Two infinitely ling straight wires A and
B ,each extending form z = –  to + , are fixed at y = – a metre and
y = +a metre respectively, as shown in the figure. If the wires A and B
each carry a current I amp into plane of the paper. Obtain the expression
for the force acting on the segment OC. What will be the force OC if
current in the wire B is reversed?

Magnetics Effect of Current [10]

Study More with www.puucho.com


www.puucho.com

EXERCISE # III
Q.1 (i) A particle of charge q and mass m moves in a circular orbit of radius r with angular speed . The ratio
of the magnitude of its magnetic moment to that of its angular momentum depends on
(A)  and q (B) , q and m (C) q and m (D)  and m
(ii) Two long parallel wires are at a distance 2d apart. They carry steady equal currents flowing out of the
plane of the paper, as shown. The variation of the magnetic field B along the XX’ is given by

(A) (B) (C) (D)

(iii) An infinitely long conductor PQR is bent to form a right angle as shown. A
current I flows through PQR. The magnetic field due to this current at the
point M is H1. Now, another infinitely long straight conductor QS is
connected at Q so that the current in PQ remaining unchanged. The magnetic
field at M is now H2. The ratio H1/H2 is given by
(A) 1/2 (B) 1 (C) 2/3 (D) 2
(iv) An ionized gas contains both positive and negative ions. If it is subjected simultaneously to an electric
field along the +x direction and a magnetic field along the +z direction, then [JEE 2000 (Scr)]
(A) positive ions deflect towards +y direction and negative ions towards y direction
(B) all ions deflect towards +y direction.
(C) all ions deflect towards y direction
(D) positive ions deflect towards y direction and negative ions towards +y direction.
Q.2 A circular loop of radius R is bent along a diameter and given a shape as
shown in the figure. One of the semicircles (KNM) lies in the x  z plane
and the other one (KLM) in the y  z plane with their centers at the origin .
Current I is flowing through each of the semicircles as shown in figure .
(i) A particle of charge q is released at the origin with a velocity v =  v0 i .
Find the instantaneous force f on the particle. Assume that space is
gravity free.
(ii) If an external uniform magnetic field B j is applied, determine the forces F1 and F2 on the semicircles
KLM and KNM due to this field and the net force F on the loop . [JEE 2000 Mains, 4 + 6]
Q.3 A current of 10A flows around a closed path in a circuit which is in
the horizontal plane as shown in the figure. The circuit consists of eight
alternating arcs of radii r1 = 0.08 m and r2 = 0.12 m. Each arc subtends
the same angle at the centre.

(a) Find the magnetic field produced by this circuit at the centre.

(b) An infinitely long straight wire carrying a current of 10A is passing through the centre of the above
circuit vertically with the direction of the current being into the plane of the circuit. What is the force
acting on the wire at the centre due to the current in the circuit? What is the force acting on the arc
AC and the straight segment CD due to the current at the centre? [JEE 2001, 5 + 5]

Magnetics Effect of Current [11]

Study More with www.puucho.com


www.puucho.com

Q.4 Two particles A and B of masses mA and mB respectively and having the same
charge are moving in a plane. A uniform magnetic field exists perpendicular to this
plane. The speeds of the particles are vA and vB respectively and the trajectories
are as shown in the figure. Then
(A) mAvA < mBvB (B) mAvA > mBvB
(C) mA < mB and vA < vB (D) mA = mB and vA = vB [JEE, 2001 (Scr)]

Q.5 A non-planar loop of conducting wire carrying a current I is placed as shown


in the figure. Each of the straight sections of the loop is of length 2a. The magnetic
field due to this loop at the point P (a, 0, a) points in the direction
1 1
(A) ( ĵ  k̂ ) (B) ( ĵ  k̂  î )
2 3
1 1
(C) (î  ĵ  k̂ ) (D) (î  k̂ ) [JEE, 2001 (Scr)]
3 2
Q.6 A coil having N turns is wound tightly in the form of a spiral with inner and outer radii a and b respectively.
When a current I passes through the coil, the magnetic field at the centre is [JEE, 2001 Screening]
 0 NI 2 0 NI  0 NI b 0I N b
(A) (B) (C) ln (D) ln
b a 2( b  a ) a 2( b  a ) a
Q.7 A particle of mass m and charge q moves with a constant velocity v along the positive x direction. It
enters a region containing a uniform magnetic field B directed along the negative z direction, extending
from x = a to x = b. The minimum value of v required so that the particle can just enter the region x > b is
(A) q b B/m (B) q( b – a) B/m (C) q a B/m (D) q(b + a) B/2m
[JEE 2002 (screening), 3]
Q.8 A long straight wire along the z-axis carries a current I in the negative z direction. The magnetic vector

field B at a point having coordinates (x, y) in the z = 0 plane is [JEE 2002 (screening), 3]
 0 I (yi  xj)  0 I (xi  yj)  0 I (xj  yi )  0 I (xi  yj )
(A) (B) (C) (D)
2 (x 2  y 2 ) 2 (x 2  y 2 ) 2  (x 2  y 2 ) 2  (x 2  y 2 )

Q.9 The magnetic field lines due to a bar magnet are correctly shown in [JEE 2002 (screening), 3]

(A) (B) (C) (D)

Q.10 A rectangular loop PQRS made from a uniform wire has length a, width
b and mass m. It is free to rotate about the arm PQ, which remains hinged
along a horizontal line taken as the y-axis (see figure). Take the vertically

upward direction as the z-axis. A uniform magnetic field B  ( 3 i  4 k ) B0
exists in the region. The loop is held in the x-y plane and a current I is
passed through it. The loop is now released and is found to stay in the
horizontal position in equilibrium.
(a) What is the direction of the current I in PQ?
(b) Find the magnetic force on the arm RS.
(c) Find the expression for I in terms of B0, a, b and m. [JEE 2002, 1+1+3]
Magnetics Effect of Current [12]

Study More with www.puucho.com


www.puucho.com

Q.11 A circular coil carrying current I is placed in a region of uniform magnetic field acting
perpendicular to a coil as shown in the figure. Mark correct option [JEE 2003 (Scr)]
(A) coil expands (B) coil contracts
(C) coil moves left (D) coil moves right

Q.12

Figure represents four positions of a current carrying coil is a magnetic field directed towards right. n̂ represent
the direction of area of vector of the coil. The correct order of potential energy is : [JEE 2003 (Scr)]
(A) I > III > II > IV (B) I < III < II < IV
(C) IV < I < II < II (D) II > II > IV > I

Q.13 A wheel of radius R having charge Q, uniformly distributed on the rim of the
wheel is free to rotate about a light horizontal rod. The rod is suspended by light
inextensible stringe and a magnetic field B is applied as shown in the figure. The
3T
initial tensions in the strings are T0. If the breaking tension of the strings are 0 ,
2
find the maximum angular velocity 0 with which the wheel can be rotate.
[JEE 2003]

Q.14 A proton and an alpha particle, after being accelerated through same potential difference, enter a uniform
magnetic field the direction of which is perpendicular to their velocities. Find the ratio of radii of the
circular paths of the two particles. [JEE 2004]

Q.15 In a moving coil galvanometer, torque on the coil can be expressed as  = ki, where i is current through
the wire and k is constant. The rectangular coil of the galvanometer having numbers of turns N, area A
and moment of inertia I is placed in magnetic field B. Find
(a) k in terms of given parameters N, I, A and B.
(b) the torsional constant of the spring, if a current i0 produces a deflection of /2 in the coil in reaching
equilibrium position.
(c) the maximum angle through which coil is deflected, id charge Q is passed through the coil almost
instantaneously. (Ignore the damping in mechanical oscillations) [JEE 2005]

Q.16 An infinite current carrying wire passes through point O and in


perpendicular to the plane containing a current carrying loop
ABCD as shown in the figure. Choose the correct option (s).
(A) Net force on the loop is zero.
(B) Net torque on the loop is zero.
(C) As seen from O, the loop rotates clockwise.
(D) As seen from O, the loop rotates anticlockwise [JEE 2006]

Magnetics Effect of Current [13]

Study More with www.puucho.com


www.puucho.com

 
Q.17 A magnetic field B = B0 ˆj exists in the region a < x < 2a and B = –B0 ˆj , in the
region 2a < x < 3a, where B0 is a positive constant. A positive point charge

moving with a velocity v = v0 î , where v0 is a positive constant, enters the
magnetic field at x = a. The trajectory of the charge in this region can be like,

(A) (B) (C) (D)

[JEE 2007]

Q.18 Two wires each carrying a steady current I are shown in four configurations in Column I. Some of the
resulting effects are described in Column II. Match the statements in Column I with the statements in
Column II and indicate your answer by darkening appropriate bubbles in the 4 × 4 matrix given in the
ORS. [JEE 2007]
Column I Column II
(A) Point P is situated (P) The magnetic fields (B) at P due to the
midway between the wires currents in the wires are in the same
direction.
(B) Point P is situated at the (Q) The magnetic fields (B) at P due to the
mid-point of the line joining currents in the wires are in opposite
the centers of the circular directions
wires, which have same radii.
(C) Point P is situated at the (R)There is no magnetic field at P
mid-point of the line joining
the centers of the circular
wires, which have same radii.
(D) Point P is situated at the (S) The wires repel each other
common center of the wires

Q.19 A particle of mass m and charge q, moving with velocity V enters Region II normal to the boundary as
shown in the figure. Region II has a uniform magnetic field B perpendicular to the plane of the paper. The
length of the Region II is . Choose the correct choice(s) [JEE 2008]
Figure
Region I Region II Region III
× × × ×
× × × ×
V × × × ×
× × × ×
× × × ×

qB
(A) The particle enters Region III only if its velocity V >
m
qB
(B) The particle enters Region III only if its velocity V <
m
qB
(C) Path length of the particle in Region II is maximum when velocity V =
m
(D) Time spent in Region II is same for any velocity V as long as the particle returns to Region I

Magnetics Effect of Current [14]

Study More with www.puucho.com


www.puucho.com

Q.20 STATEMENT-1 : The sensitivity of a moving coil galvanometer is increased by placing a suitable
magnetic material as a core inside the coil.
and
STATEMENT-2 : Soft iron has a high magnetic permeability and cannot be easily magnetized or
demagnetized. [JEE 2008]
(A) STATEMENT-1 is True, STATEMENT-2 is True ; STATEMENT-2 is a correct explanation for
STATEMENT-1
(B) STATEMENT-1 is True, STATEMENT-2 is True ; STATEMENT-2 is NOT a correct explanation
for STATEMENT-1
(C) STATEMENT-1 is True, STATEMENT-2 is False
(D) STATEMENT-1 is False, STATEMENT-2 is True

Q.21 Six point charges, each of the same magnitude q, are arranged in different manners as shown in Column
II. In each case, a point M and a line PQ passing through M are shown. Let E be the electric field and V
be the electric potential at.M (poteptial at infinity is zero) due to the given charge distribution when it is at
rest. Now, the whole system is set into rotation with a constant angular velocity about the line PQ. Let B
be the magnetic field at M and µ be the magnetic moment of the system in this condition. Assume each
rotating charge to be equivalent to a steady current. [JEE-2009]
Column-I + –
Q Column-I
(A) E=0 (p) Charges are at the corners of a regular
¯ + hexagon. M isthe centre of the hexagon.
M PQ is perpendicular to the plane of the

hexagon.
P +

P
(B) V0 (q) Charges are on a line perpendicular to
– + – + – + PQ at equal intervals. M is the mid-
M point between the two innermost
charges.
Q

(C) B=0 (r) – Q Charges are placed at the corners of a


+ + rectangle of sides a and 2a and at the
– M – mid points of the longer sides. M is at
the centre of the rectangle. PQ is parallel
P to the longer sides.
+
(D) µ0 (s) – + – Charges are placed at the corners of a
rectangle of sides a and 2a and at the
M
P Q mid points of the longer sides. M is at
the centre of the rectangle. PQ is parallel
– + –
to the longer sides.

(t) P Charges are placed on two coplanar,


+ –
identical insulating rings at equal
intervals. M is the mid-point btween the
+ M
+ – – centres of the rings. PQ is
Q
perpendicular to the line joining the
centres and coplanar to the rings.

Magnetics Effect of Current [15]

Study More with www.puucho.com


www.puucho.com

Q.22 Column II shows five systems in which two objects are labelled as X and Y. Also in each case a point
P is shown. Column I gives some statements about X and/or Y. Match these statements to the appropriate
system(s) from Column II. [JEE-2009]
Column-I Column-II
(A) The force exerted (p) Y Block Y of mass M left on a fixed
by X on Y has a X inclined plane X, slides on it with a
magnitude Mg. constant velocity.
P

(B) The gravitational (q) P Two ring magnets Y and Z, each of


potential energy mass M, are kept in frictionless vertical
of X is continuously Z plastic stand so that they repel each
increasing Y other. Y rests on the base X and Z
hangs in air in equilibrium. P is the
X
topmost point of the stand on the
common axis of the two rings. The
whole system is in a lift that is going up
with a constant elocity.

(C) Mechanical energy (r) A pulley Y of mass m0 is fixed to a


of the system X + Y P Y
table through a clamp X. A block of
is continuously mass M hangs from a string that goes
decreasing X over the pulley and is fixed at point P
of the table. The whole system is kept
in a lift that is going down with a constant
velocity.

(D) The torque of the (s) A sphere Y of mass M is put in a


Y
weight of Y about nonviscous liquid X kept in a container
point P is zero. X at rest. The sphere is released and it
P moves down in the liquid.

(t) A sphere Y of mass M is falling with


Y itsterminal velocity in a viscous liquid
X kept in a container.
X
P

Magnetics Effect of Current [16]

Study More with www.puucho.com


www.puucho.com

OBJECTIVE QUESTION BANK


ONLY ONE OPTION IS CORRECT.
Take approx. 2 minutes for answering each question.
Q.1 A current of i ampere is flowing through each of the bent wires as shown the magnitude and direction of
magnetic field at 0 is
 0i  1 2   0i  1 3 
(A)    (B)   
4  R R  4  R R 
 0i  1 3   0i  1 3 
(C)    (D)   
8  R 2R   8  R R 
Q.2 Net magnetic field at the centre of the circle O due to a current carrying
loop as shown in figure is ( < 180°)
(A) zero (B) perpendicular to paper inwards
(C) perpendicular to paper outwards
(D) is perpendicular to paper inwards if   90° and perpendicular to paper outwards if 90°<180°
Q.3 A charge particle A of charge q = 2 C has velocity v = 100 m/s. When it passes through
point A and has velocity in the direction shown. The strength of magnetic field at point B
due to this moving charge is (r = 2 m).
(A) 2.5 T (B) 5.0 T (C) 2.0 T (D) None

Q.4 Three rings, each having equal radius R, are placed mutually perpendicular to
each other and each having its centre at the origin of co-ordinate system. If
current I is flowing thriugh each ring then the magnitude of the magnetic field at
the common centre is

(A) 3
0I
2R
(B) zero 
(C) 2  1 
0I
2R

(D) 3  2 
0I
2R
Q.5 Two concentric coils X and Y of radii 16 cm and 10 cm lie in the same vertical plane containing N-S
direction. X has 20 turns and carries 16 A. Y has 25 turns & carries 18A. X has current in anticlockwise
direction and Y has current in clockwise direction for an observer, looking at the coils facing the west.
The magnitude of net magnetic field at their common centre is
(A) 5 × 10–4 T towards west (B) 13 × 10–4 T towards east
(C) 13 × 10–4 T towards west (D) 5 × 10–4 T towards east

Q.6 The dimension of where  is permeability &  is permittivity is same as :

(A) Resistance (B) Inductance (C) Capacitance (D) None of these
Q.7 A current I flows around a closed path in the horizontal plane of the circle as
shown in the figure. The path consists of eight arcs with alternating radii r and 2r.
Each segment of arc subtends equal angle at the common centre P. The magnetic
field produced by current path at point P is
3 0I
(A) ; perpendicular to the plane of the paper and directed inward.
8 r
3 0I
(B) ; perpendicular to the plane of the paper and directed outward.
8 r
1 0I
(C) ; perpendicular to the plane of the paper and directed inward.
8 r
1 0I
(D) ; perpendicular to the plane of the paper and directed outward..
8 r
Magnetics Effect of Current [17]

Study More with www.puucho.com


www.puucho.com

Q.8 Infinite number of straight wires each carrying current I are equally
placed as shown in the figure. Adjacent wires have current in
opposite direction. Net magnetic field at point P is
 0 I ln 2  0 I ln 4
(A) k̂ (B) k̂
4 3 a 4 3 a
 0 I ln 4
(C) (k̂ ) (D) Zero
4 3 a

Q.9 Two mutually perpendicular conductors carrying currents I1 and I2 lie in one plane. Locus of the point at
which the magnetic induction is zero, is a
(A) circle with centre as the point of intersection of the conductor.
(B) parabola with vertex as the point of intersection of the conductors
(C) straight line passing through the point of intersection of the conductors.
(D) rectangular hyperbola

Q.10 Find the magnetic field at P due to the arrangement shown


 0i  1  2 0i  0i  0i  1 
(A) 1    (B)  (C)  (D) 1  
2 d  2 2 d 2 d 2 d  2

Q.11 Equal current i is flowing in three infinitely long wires along positive x, y and z directions. The magnetic
field at a point (0, 0, –a) would be:
 0i  0i  0i  0i
(A) ( ĵ  î ) (B) ( î  ĵ) (C) ( î  ĵ) (D) (î  ĵ  k̂ )
2a 2a 2a 2a

Q.12 Two very long straight parallel wires, parallel to y-axis, carry currents 4I and I, along +y direction and –y direction,
respectively. The wires are passes through the x-axis at the points (d, 0, 0) and (– d, 0, 0) respectively. The graph
of magnetic field z-component as one moves along the x-axis from x = – d to x = +d, is best given by

(A) (B) (C) (D)

Q.13 A long thin walled pipe of radius R carries a current I along its length. The current
density is uniform over the circumference of the pipe. The magnetic field at the center
of the pipe due to quarter portion of the pipe shown, is
0I 2 0I 2 0 I 2
(A) (B) 2 (C) (D) None
4 2 R  R 2 R

Q.14 A hollow cylinder having infinite length and carrying uniform current per unit length 
along the circumference as shown. Magnetic field inside the cylinder is
0
(A) (B) 0 (C) 20 (D) none
2

Magnetics Effect of Current [18]

Study More with www.puucho.com


www.puucho.com

Q.15 Two long conductors are arranged as shown above to form overlapping
cylinders, each of raidus r, whose centers are separated by a distance
d. Current of density J flows into the plane of the page along the shaded
part of one conductor and an equal current flows out of the plane of the
page along the shaded portion of the other, as shown. What are the
magnitude and direction of the magnetic field at point A?
(A) (0/2)dJ, in the +y-direction (B) (0/2)d2/r, in the +y-direction
(C) (0/2)4d2J/r, in the –y-direction (D) (0/2)Jr2/d, in the –y-direction
(E) There is no magnetic field at A.

Q.16 An electron is moving along positive x-axis. A uniform electric field exists towards negative y-axis. What
should be the direction of magnetic field of suitable magnitude so that net force of electron is zero
(A) positive z- axis (B) negative z-axis (C) positive y-axis (D) negative y-axis

Q.17 A particle of charge q and mass m starts moving from the origin under the action of an electric field
  
E  E 0 î and B  B0 î with velocity v  v0 ĵ . The speed of the particle will become 2v0 after a time
2 m v0 2 Bq 3 Bq 3 mv0
(A) t = (B) t = mv (C) t = (D) t =
qE 0 mv0 qE

Q.18 An electron is projected with velocity v0 in a uniform electric field E perpendicular to the field. Again it is
projetced with velocity v0 perpendicular to a uniform magnetic field B/ If r1 is initial radius of curvature
just after entering in the electric field and r2 is initial radius of curvature just after entering in magnetic field
then the ratio r1 r2 is equal to
Bv 02 B Ev 0 Bv 0
(A) (B) (C) (D)
E E B E

Q.19 A uniform magnetic field B  B0 ĵ exists in a space. A particle of mass m and charge q is projected
towards negative x-axis with speed v from the a point (d, 0, 0). The maximum value v for which the
particle does not hit y-z plane is
2 Bq Bqd Bq Bqd
(A) (B) (C) (D)
dm m 2dm 2m

Q.20 Two protons move parallel to each other, keeping distance r between them, both moving with same

velocity V . Then the ratio of the electric and magnetic force of interaction between them is
(A) c 2 V 2 (B) 2c 2 V 2 (C) c 2 2V 2 (D) None

Q.21 Three ions H+, He+ and O+2 having same kinetic energy pass through a region in which there width is a
uniform magnetic field perpendicular to their velocity, then :
(A) H+ will be least deflected. (B) He+ and O+2 will be deflected equally.
(C) O+2 will be deflected most. (D) all will be deflected equally.

Q.22 An electron having kinetic energy T is moving in a circular orbit of radius R perpendicular to a uniform

magnetic induction B . If kinetic energy is doubled and magnetic induction tripled, the radius will become
3R 3 2 4
(A) (B) R (C) R (D) R
2 2 9 3

Magnetics Effect of Current [19]

Study More with www.puucho.com


www.puucho.com

Q.23 An electron (mass = 9.1 × 1031 ; charge =  1.6 × 1019 C) experiences no deflection if subjected to
an electric field of 3.2 × 105 V/m and a magnetic field of 2.0 × 103 Wb/m2 . Both the fields are normal
to the path of electron and to each other . If the electric field is removed, then the electron will revolve in
an orbit of radius :
(A) 45 m (B) 4.5 m (C) 0.45 m (D) 0.045 m
 
Q.24 A charged particle moves in a magnetic field B  10 î with initial velocity u  5î  4ˆj . The path of the
particle will be
(A) straight line (B) circle (C) helical (D) none

 
Q.25 A electron experiences a force 4.0 î  3.0 ˆj × 10–13 N in a uniform magnetic field when its velocity is
 
2.5 k̂ 107 ms–1. When the velocity is redirected and becomes 1.5 î  2.0 ĵ  10 7 ms–1, the magnetic

force of the electron is zero. The magnetic field vector B is :
(A) – 0.075 î  0.1 ˆj (B) 0.1 î  0.075 ˆj (C) 0.075 î  0.1 ˆj  k̂ (D) 0.075 î  0.1 ˆj

Q.26 Electrons moving with different speeds enter a uniform magnetic field in a direction perpendicular to the
field. They will move along circular paths.
(A) of same radius
(B) with larger radii for the faster electrons
(C) with smaller radii for the faster electrons
(D) either (B) or (C) depending on the magnitude of the magnetic field

Q.27 In the previous question, time periods of rotation will be :


(A) same for all electrons
(B) greater for the faster electrons
(C) smaller for the faster electrons
(D) either (B) or (C) depending on the magnitude of the magnetic field

Q.28 OABC is a current carrying square loop an electron is projected from the centre of loop along its
diagonal AC as shown. Unit vector in the direction of initial acceleration will be
 î  ĵ 
(A) k̂ 
(B)  
2 
 
î  ĵ
(C) – k̂ (D)
2

Q.29 A particle having charge of 1 C, mass 1 kg and speed 1 m/s enters a uniform magnetic field, having
magnetic induction of 1 T, at an angle  = 30° between velocity vector and magnetic induction. The pitch
of its helical path is (in meters)
3 
(A) (B) 3 (C) (D) 
2 2

Q.30 A charged particle is released from rest in a region of uniform electric and magnetic fields, which are
parallel to each other. The locus of the particle will be
(A) helix of constant pitch (B) straight line
(C) helix of varying pitch (D) cycloid

Magnetics Effect of Current [20]

Study More with www.puucho.com


www.puucho.com

Q.31 A particle of specific charge (charge/mass)  starts moving from the origin under the action of an electric
 
field E  E 0 î and magnetic field B  B0 k̂ . Its velocity at (x0, y0, 0) is ( 4î  3ˆj) . The value of x0 is:

13 E 0 16 B0 25 5
(A) 2 B (B) E0 (C) 2E (D) 2B
0 0 0

Q.32 A particle of specific charge (q/m) is projected from the origin of coordinates with initial velocity
[ui – vj]. Uniform electric magnetic fields exist in the region along the +y direction, of magnitude E and B.
The particle will definitely return to the origin once if
(A) [ vB 2E] is an integer (B) (u2 + v2)1/2 [B E] is an integer
(C) [ vB E] in an integer (D) [uB E] is an integer

 
Q.33 An electron moving with a velocity V1  2 î m/s at a point in a magnetic field experiences a force F1  2 ĵN .
 
If the electron is moving with a velocity V2  2 ĵ m/s at the same point, it experiences a force F2  2 î N .

The force the electron would experience if it were moving with a velocity V3  2k̂ m/s at the same point is

(A) zero (B) 2k̂N (C)  2k̂N (D) information is insufficient

Q.34 Two particles of charges +Q and –Q are projected from the same point with a velocity v in a region of
uniform magnetic field B such that the velocity vector makes an angle  with the magnetic field. Their
masses are M and 2M, respectively. Then, they will meet again for the first time at a point whose
distance from the point of projection is
(A) 2Mv cos  QB (B) 8Mv cos  QB (C) Mv cos  QB (D) 4Mv cos  QB

Q.35 A particle with charge +Q and mass m enters a magnetic field of magnitude B,
existing only to the right of the boundary YZ. The direction of the motion of the
m
particle is perpendicular to the direction of B. Let T = 2 . The time spent
QB
by the particle in the field will be
   2     2 
(A) T (B) 2T (C) T   (D) T  
 2   2 

Q.36 In the previous question, if the particle has –Q charge, the time spend by the particle in the field will be
   2     2 
(A) T (B) 2T (C) T   (D) T  
 2   2 

Q.37 The direction of magnetic force on the electron as shown in the diagram is along
(A) y-axis (B) –y-axis
(C) z-axis (D) –z-axis

Magnetics Effect of Current [21]

Study More with www.puucho.com


www.puucho.com

Q.38 A block of mass m & charge q is released on a long smooth inclined plane
magnetic field B is constant, uniform, horizontal and parallel to surface as
shown. Find the time from start when block loses contact with the surface.
m cos  m cos ec  m cot 
(A) (B) (C) (D) none
qB qB qB

Q.39 A straight rod of mass m and length L is suspended from the identical spring as shown in the figure. The
spring stretched by a distance of x0 due to the weight of the wire. The circuit has total resistance R.
When the magnetic field perpendicular to the plane of the paper is switched on, springs are observed to
extend further by the same distance. The magnetic field strength is
mgR
(A) ; directed outward from the plane of the paper
L
mgR
(B) ; directed outward from the plane of the paper
2  x0
mgR
(C) ; directed into the plane of the paper
L
mgR
(D) ; directed into the plane of the paper
 x0

Q.40 A conducting wire bent in the form of a parabola y2 = 2x carries a current


i = 2 A as shown in figure. This wire is placed in a uniform magnetic field

B  4 k̂ Tesla. The magnetic force on the wire is (in newton)
(A)  16 î (B) 32 î (C)  32 î (D) 16 î

Q.41 A semi circular current carrying wire having radius R is placed in


x-y plane with its centre at origin ‘O’. There is non-uniform magnetic
 B x
field B  o k̂ (here Bo is +ve constant) is existing in the region. The
2R
magnetic force acting on semi circular wire will be along
(A) – x-axis (B) + y-axis
(C) – y-axis (D) + x-axis

Q.42 A square loop ABCD, carrying a current i, is placed near and coplanar with a long straight conductor
XY carrying a current I, the net force on the loop will be
2 0 Ii  0 Ii 2 0 Iil  0 Iil
(A) (B) (C) (D)
3 2 3 2

Q.43 A metal ring of radius r = 0.5 m with its plane normal to a uniform magnetic field B of induction 0.2 T
carries a current I = 100 A. The tension in newtons developed in the ring is:
(A) 100 (B) 50 (C) 25 (D) 10

Magnetics Effect of Current [22]

Study More with www.puucho.com


www.puucho.com

Q.44 In given figure, X and Y are two long straight parallel conductors each carrying
a current of 2 A. The force on each conductor is F newtons. When the current
in each is changed to 1 A and reversed in direction, the force on each is now
(A) F/4 and unchanged in direction (B) F/2 and reversed in direction
(C) F/2 and unchanged in direction (D) F/4 and reversed in direction

Q.45 A conducting ring of mass 2 kg and radius 0.5 m is placed on a smooth horizontal
plane. The ring carries a current i = 4A. A horizontal magnetic field B = 10T is
switched on at time t = 0 as shown in figure. The initial angular acceleration of
the ring will be
(A) 40  rad/s2 (B) 20  rad/s2 (C) 5  rad/s2 (D) 15  rad/s2

Q.46 In the figure shown a coil of single turn is wound on a sphere of radius R and mass
m. The plane of the coil is parallel to the plane and lies in the equatorial plane of
the sphere. Current in the coil is i. The value of B if the sphere is in equilibrium is
mg cos  mg mg tan  mg sin 
(A) (B) (C) (D)
iR iR iR iR

Q.47 The magnetic moment of a circular orbit of radius ‘r’ carrying a charge ‘q’ and rotating with velocity v is
given by
qvr qvr
(A) (B) (C) qvr (D) qvr2
2 2

Q.48 A thin non conducting disc of radius R is rotating clockwise (see figure) with an angular velocity w about
its central axis, which is perpendicular to its plane. Both its surfaces carry +ve charges of uniform surface
density. Half the disc is in a region of a uniform, unidirectional magnetic field B parallel to the plane of the
disc, as shown. Then,
(A) The net torque on the disc is zero.
(B) The net torque vector on the disc is directed leftwards.
(C) The net torque vector on the disc is directed rightwards.
(D) The net torque vector on the disc is parallel to B.

Q.49 A rectangular coil PQ has 2n turns, an area 2a and carries a current 2I, (refer
figure). The plane of the coil is at 60° to a horizontal uniform magnetic field of
flux density B. The torque on the coil due to magnetic force is
(A) BnaI sin60° (B) 8BnaI cos60° (C) 4naI Bsin60° (D) none

Q.50 A straight current carrying conductor is placed in such a way that the current in the conductor flows in the
direction out of the plane of the paper. The
conductor is placed between two poles of two magnets, as shown.
The conductor will experience a force in the direction towards
(A) P (B) Q (C) R (D) S

Magnetics Effect of Current [23]

Study More with www.puucho.com


www.puucho.com

Q.51 Figure shows a square current carrying loop ABCD of side 10 cm and

current i = 10A. The magnetic moment M of the loop is
 
(A) (0.05) î  3k̂ A  m 2  
(B) (0.05) ˆj  k̂ A  m 2
(C) (0.05)  3î  k̂ A  m 2
 
(D) î  k̂ A  m 2

Q.52 A direct current flows in a solenoid of length L and radius R, (L >> R), producing a magnetic field of
magnitude B0 inside the solenoid.

P B0

R

Which of the following diagrams best discribes the magnetic field lines in the vicinity of P at the end of the
coil

(A) (B)

(C) (D)

Magnetics Effect of Current [24]

Study More with www.puucho.com


www.puucho.com

ASSERTION AND REASON

Q.1 Statement-1 : It is not possible for a charged particle to move in a circular path around a long straight
uncharged conductor carrying current under the influence of its magnetic field alone.
Statement-2 : The magnetic force (if nonzero) on a moving charged particle is normal to its velocity.
(A) Statement-1 is true, statement-2 is true and statement-2 is correct explanation for statement-1.
(B) Statement-1 is true, statement-2 is true and statement-2 is NOT the correct explanation for statement-1.
(C) Statement-1 is true, statement-2 is false.
(D) Statement-1 is false, statement-2 is true.

Q.2 Statement-1 : For a charged particle to pass through a uniform electro-magnetic field without change
in velocity, its velocity vector must be perpendicular to the magnetic field.
   
Statement-2 : Net Lorentz force on the particle is given by F  q[ E  v  B]
(A) Statement-1 is true, statement-2 is true and statement-2 is correct explanation for statement-1.
(B) Statement-1 is true, statement-2 is true and statement-2 is NOT the correct explanation for statement-1.
(C) Statement-1 is true, statement-2 is false.
(D) Statement-1 is false, statement-2 is true.

Q.3 Statement-1 : Ampere law can be used to find magnetic field due to finite length of a straight current
carrying wire.
Statement-2 : The magnetic field due to finite length of a straight current carrying wire is symmetric
about the wire.
(A) Statement-1 is true, statement-2 is true and statement-2 is correct explanation for statement-1.
(B) Statement-1 is true, statement-2 is true and statement-2 is NOT the correct explanation for statement-1.
(C) Statement-1 is true, statement-2 is false.
(D) Statement-1 is false, statement-2 is true.

Q.4 Statement-1 : A pendulum made of a non-conducting rigid massless rod of length  is attached to a
small sphere of mass m and charge q. The pendulum is undergoing oscillations of small

amplitude having time period T. Now a uniform horizontal magnetic field B out of plane
of page is switched on. As a result of this change, the time period of oscillations will
change.

B

m,q

Statement-2 : In the situation of statement-1, after the magnetic field is switched on the tension in string
will change (except when the bob is at extreme position)
(A) Statement-1 is true, statement-2 is true and statement-2 is correct explanation for statement-1.
(B) Statement-1 is true, statement-2 is true and statement-2 is NOT the correct explanation for statement-1.
(C) Statement-1 is true, statement-2 is false.
(D) Statement-1 is false, statement-2 is true.

Magnetics Effect of Current [25]

Study More with www.puucho.com


www.puucho.com

Q.5 Statement-1 : Net force on a current carrying loop in a non-uniform magnetic field must be
non-zero.
Statement-2 : Force on a current carrying wire of length dl placed in magnetic field B is given by
dF  idl  B
(A) Statement-1 is true, statement-2 is true and statement-2 is correct explanation for statement-1.
(B) Statement-1 is true, statement-2 is true and statement-2 is NOT the correct explanation for statement-1.
(C) Statement-1 is true, statement-2 is false.
(D) Statement-1 is false, statement-2 is true.

Q.6 Statement-1 : Two long parallel conductors carrying currents in the same direction experience a force
of attraction.
Statement-2 : The magnetic fields produced in the space between the conductors are in the same
direction.
(A) Statement-1 is true, statement-2 is true and statement-2 is correct explanation for statement-1.
(B) Statement-1 is true, statement-2 is true and statement-2 is NOT the correct explanation for statement-1.
(C) Statement-1 is true, statement-2 is false.
(D) Statement-1 is false, statement-2 is true.

Q.7 Statement-1 A charged particle can never move along a magnetic field line in absence of any other force.
  
Statement-2 Force due to magnetic field is given by F  q ( v  B)
(A) Statement-1 is true, statement-2 is true and statement-2 is correct explanation for statement-1.
(B) Statement-1 is true, statement-2 is true and statement-2 is NOT the correct explanation for statement-1.
(C) Statement-1 is true, statement-2 is false.
(D) Statement-1 is false, statement-2 is true.

ONE OR MORE THAN ONE OPTION MAY BE CORRECT


Take approx. 3 minutes for answering each question.
Q.1 In the following hexagons, made up of two different material P and Q, current enters and leaves from
points X and Y respectively. In which case the magnetic field at its centre is not zero.

(A) (B) (C) (D)

Q.2 Consider the magnetic field produced by a finitely long current carrying wire.
(A) the lines of field will be concentric circles with centres on the wire.
(B) There can be two points in the same plane where magnetic fields are same.
(C) There can be large number of points where the magnetic field is same.
(D) The magnetic field at a point is inversally proportional to the distance of the point from the wire.

Magnetics Effect of Current [26]

Study More with www.puucho.com


www.puucho.com

l
Q.3 Consider three quantities x = E/B, y = 1 /  0  0 and z = . Here, l is the length of a wire, C is a
CR
capacitance and R is a resistance. All other symbols have standard meanings.
(A) x, y have the same dimensions
(B) y, z have the same dimensions
(C) z, x have the same dimensions
(D) none of the three pairs have the same dimensions.

Q.4 Two long thin, parallel conductors carrying equal currents in the
same direction are fixed parallel to the x-axis, one passing through
y = a and the other through y = –a. The resultant magnetic field
due to the two conductors at any point is B. Which of the
following are correct?
(A) B = 0 for all points on the x-axis
(B) At all points on the y-axis, excluding the origin, B has only a z-component.
(C) At all points on the z-axis, excluding the origin, B has only a y-component.
(D) B cannot have an x-component.

Q.5 Current flows through uniform, square frames as shown. In which case is the magnetic field at the centre
of the frame not zero?

(A) (B) (C) (D)

Q.6 A long straight wire carries a current along the x-axis. Consider the points A(0, 1, 0), B(0, 1, 1),
C(1, 0, 1) and D(1, 1, 1). Which of the following pairs of points will have magnetic fields of the same
magnitude?
(A) A and B (B) A and C (C) B and C (D) B and D

Q.7 In the previous question, if the current is i and the magnetic field at D has magnitude B,
 0i  0i
(A) B = (B) B =
2 2 2 3
(C) B is parallel to the x-axis (D) B makes an angle of 45° with the xy plane

Q.8 Which of the following statement is correct :


(A) A charged particle enters a region of uniform magnetic field at an angle 850 to magnetic lines of force.
The path of the particle is a circle.
(B) An electron and proton are moving with the same kinetic energy along the same direction. When they
pass through uniform magnetic field perpendicular to their direction of motion, they describe circular
path.
(C) There is no change in the energy of a charged particle moving in a magnetic field although magnetic
force acts on it.
(D) Two electrons enter with the same speed but in opposite direction in a uniform transverse magnetic
field. Then the two describe circle of the same radius and these move in the same direction.

Magnetics Effect of Current [27]

Study More with www.puucho.com


www.puucho.com

Q.9 Two identical charged particles enter a uniform magnetic field with same speed but at angles 30° and 60°
with field. Let a, b and c be the ratio of their time periods, radii and pitches of the helical paths than
(A) abc = 1 (B) abc > 1 (C) abc < 1 (D) a = bc

Q.10 Consider the following statements regarding a charged particle in a magnetic field. Which of the statements are true:
(A) Starting with zero velocity, it accelerates in a direction perpendicular to the magnetic field.
(B) While deflecting in magnetic field its energy gradually increases .
(C) Only the component of magnetic field perpendicular to the direction of motion of the charged
particle is effective in deflecting it.
(D) Direction of deflecting force on the moving charged particle is perpendicular to its velocity.

Q.11 An electron is moving along the positive X-axis. You want to apply a magnetic field for a short time so
that the electron may reverse its direction and move parallel to the negative Xaxis. This can be done by
applying the magnetic field along
(A) Y-axis (B) Z-axis (C) Y-axis only (D) Z-axis only

Q.12 In a region of space, a uniform magnetic field B exists in the y-direction. A proton
is fired from the origin, with its initial velocity v making a small angle  with the
y-direction in the yz plane. In the subsequent motion of the proton,
(A) its x-coordinate can never be positive
(B) its x- and z-coordinates cannot both be zero at the same time
(C) its z-coordinate can never be negative
(D) its y-coordinate will be proportional to the square of its time of flight

Magnetics Effect of Current [28]

Study More with www.puucho.com


www.puucho.com

ANSWER KEY
EXERCISE # I
Q.1 In the plane of the drawing from right to left

Q.2 (a) 0 (b) 1.41 × 10 –6 T , 45º in xz - plane, (c) 5 × 10 –6 T , + x - direction]


2  0i 5 (2 2  1)  0i
Q.3 Q.4 4 × 10–5 T Q.5
8 l 2 2 a

0I  3 1  0 i  3 
Q.6 zero Q.7  k̂  ĵ  Q.8    1
4R  4   4r  2 

0 i
Q.9 B=
4R

2 2 2  2   1  Q.10 (i) 1.3 ×10–4T, (ii) zero

 0 br12  0 bR 3  0iqv
Q.11 0 weber.m–1 Q.12 B1 = , B2 = Q.13
3 3r2 2 a

2mv 0 mEI
Q.14 10 k̂ Q.15 Q.16 Q.17 zero
qB Be

3mv 2 3mv 3
Q.18 2 IRB Q.19 F = a2i ˆj Q.20 (a) , (b) , (c) zero
4qa 4a

m  0 I I C  1 1 
Q.21 T0 = 2  = 0.57 s Q.22  to the left
6IB 2  a b 

d I  Q 2
Q.23 z = 0 , x = ± , (ii) Q.24 h tan 2 
3  d  4

EXERCISE # II
 0  4I   0  I 2  1
Q.1 (i)   along Y-axis, (ii) 4  2a  10 , tan 4     with positive axis
4  a     3

QV  0 I  3 3    3 2
Q.2 (a) m 6a   1 , (b) BI  a ĵ Q.3 (a) 20 min. (b) 5.94 x 10–2 Nm
  3 4 
Q.4 15 C

Q.5 (a) 1.4 x 104 m/s (b) 4.5 x 1023 N (down) (c) 2.8 x 104 V/m (down)
(d) 5.7 x 106 V (top + , bottom ) (e) same as (c)

mg mg  0 iJ 0  d 
Q.6 (a) I = (b) I = Q.7 tan 1 (k̂ )
 
1/ 2
r B2x  B2y  r Bx   2h 

Magnetics Effect of Current [29]

Study More with www.puucho.com


www.puucho.com

Q.8 0.62 N < F < 0.88 N Q.9 Bmin = 4.7×10–3 T

BI 0 L2
Q.10 i1 = 0.1110 A, i2 = 0.096 A Q.11

(a)  =
2
 
î ˆj (b) q =
3 BI 0
4 M
Dt2

 0 I1I 2   I   L  a  
Q.12
4
ln (3) along – ve z direction Q.14 F=   n 
     a 
 
 k̂ , zero

EXERCISE # III
Q.1 (i) C (ii) B (iii) C (iv) C

0 I
Q.2 (i)  4R q v0 k̂ (ii) F1 = 2 I R B F2 =2 I R B , Net force = F1 + F2 = 4 I R B i

Q.3 (a) 6.6 × 10–5T, (b) 0, 0, 8 × 10–6Nt


Q.4 B Q.5 D Q.6 C Q.7 B Q.8 A Q.9 D

 mg
Q.10 (a) current in loop PQRS is clockwise from P to QRS., (b) F = BI0b (3k̂4î) , (c) I =
6bB0

d T0 rp mp q 1
Q.11 A Q.12 A Q.13  = Q.14  . 
QR 2 B r m q p 2

2i 0 NAB NAB
Q.15 (a) k = NAB, (b) C = , (c) Q × 2li 0 Q.16 A, C Q.17 A

Q.18 (A) Q, R; (B) P; (C) Q, R; (D) Q or (A) Q,R; (B) P; (C) Q,R; (D) Q,S

Q.19 A, C, D Q.20 C Q.21 (A) p,r,s (B) r, s (C) p,q,t (D) r,s

Q.22 (A) p, t (B) q,s,t (C) p,r,t (D) q

Magnetics Effect of Current [30]

Study More with www.puucho.com


www.puucho.com

OBJECTIVE QUESTION BANK


ONLY ONE OPTION IS CORRECT.
Q.1 D Q.2 C Q.3 A Q.4 A Q.5 A Q.6 A Q.7 A
Q.8 B Q.9 C Q.10 A Q.11 A Q.12 C Q.13 A Q.14 B

Q.15 A Q.16 B Q.17 D Q.18 D Q.19 B Q.20 A Q.21 B


Q.22 C Q.23 C Q.24 C Q.25 A Q.26 B Q.27 A Q.28 B
Q.29 B Q.30 B Q.31 C Q.32 C Q.33 A Q.34 D Q.35 C
Q.36 D Q.37 A Q.38 C Q.39 A Q.40 B Q.41 A Q.42 A

Q.43 D Q.44 A Q.45 A Q.46 B Q.47 B Q.48 B Q.49 B


Q.50 B Q.51 A Q.52 C

ASSERTION AND REASON


Q.1 B Q.2 D Q.3 D Q.4 D Q.5 D Q.6 C Q.7 D
ONE OR MORE THAN ONE OPTION MAY BE CORRECT
Q.1 A Q.2 A, B, C Q.3 A, B, C Q.4 A, B, C, D
Q.5 C Q.6 B, C Q.7 A, D Q.8 B, C
Q.9 A, D Q.10 C, D Q.11 A, B Q.12 A

Magnetics Effect of Current [31]

Study More with www.puucho.com

You might also like